Solve for $z$, which satisfy $arg(z-3-2i) = frac{pi}{6}$ and $arg(z-3-4i) = frac{2pi}{3}$.












2














solve for $z$, which satisfy $displaystyle arg(z-3-2i) = frac{pi
}{6}$
and $displaystylearg(z-3-4i) = frac{2pi}{3}$.



So I'm first assuming $z=x+iy$, then putting it in the place of $z$ and putting real parts together and imaginary parts together.



Then I'm using $tantheta = frac{text{imaginary part}}{text{real part}}$



$z-3-4i = (x-3)+(y-2)i$



Then,



$tan30° = frac{y-2}{x-3}$



$frac{1}{sqrt3} = frac{y-2}{x-3}$



I'm processing like this and my answer comes as $y=5/2$



And $x= 3+frac{5sqrt3}{2}-2sqrt3$



But the answer key says there's no such $z$ which satisfies this equation.
Is this the wrong way to solve this question, is my answer wrong or the answer key's?










share|cite|improve this question
























  • Why are you equating the arguments of the $Arg$'s? Or should that $2$ be a $4$?
    – Michael Burr
    Nov 22 '18 at 4:00






  • 1




    It may be useful to draw a diagram.
    – Kemono Chen
    Nov 22 '18 at 4:04










  • @MichaelBurr i didn't get you. I'm not equating the arguments.
    – Kaustuv Sawarn
    Nov 22 '18 at 4:06










  • @KemonoChen but I get the answer this way, i want to know what am I doing wrong?
    – Kaustuv Sawarn
    Nov 22 '18 at 4:07










  • Your approach is wrong .
    – Akash Roy
    Nov 22 '18 at 4:07
















2














solve for $z$, which satisfy $displaystyle arg(z-3-2i) = frac{pi
}{6}$
and $displaystylearg(z-3-4i) = frac{2pi}{3}$.



So I'm first assuming $z=x+iy$, then putting it in the place of $z$ and putting real parts together and imaginary parts together.



Then I'm using $tantheta = frac{text{imaginary part}}{text{real part}}$



$z-3-4i = (x-3)+(y-2)i$



Then,



$tan30° = frac{y-2}{x-3}$



$frac{1}{sqrt3} = frac{y-2}{x-3}$



I'm processing like this and my answer comes as $y=5/2$



And $x= 3+frac{5sqrt3}{2}-2sqrt3$



But the answer key says there's no such $z$ which satisfies this equation.
Is this the wrong way to solve this question, is my answer wrong or the answer key's?










share|cite|improve this question
























  • Why are you equating the arguments of the $Arg$'s? Or should that $2$ be a $4$?
    – Michael Burr
    Nov 22 '18 at 4:00






  • 1




    It may be useful to draw a diagram.
    – Kemono Chen
    Nov 22 '18 at 4:04










  • @MichaelBurr i didn't get you. I'm not equating the arguments.
    – Kaustuv Sawarn
    Nov 22 '18 at 4:06










  • @KemonoChen but I get the answer this way, i want to know what am I doing wrong?
    – Kaustuv Sawarn
    Nov 22 '18 at 4:07










  • Your approach is wrong .
    – Akash Roy
    Nov 22 '18 at 4:07














2












2








2


0





solve for $z$, which satisfy $displaystyle arg(z-3-2i) = frac{pi
}{6}$
and $displaystylearg(z-3-4i) = frac{2pi}{3}$.



So I'm first assuming $z=x+iy$, then putting it in the place of $z$ and putting real parts together and imaginary parts together.



Then I'm using $tantheta = frac{text{imaginary part}}{text{real part}}$



$z-3-4i = (x-3)+(y-2)i$



Then,



$tan30° = frac{y-2}{x-3}$



$frac{1}{sqrt3} = frac{y-2}{x-3}$



I'm processing like this and my answer comes as $y=5/2$



And $x= 3+frac{5sqrt3}{2}-2sqrt3$



But the answer key says there's no such $z$ which satisfies this equation.
Is this the wrong way to solve this question, is my answer wrong or the answer key's?










share|cite|improve this question















solve for $z$, which satisfy $displaystyle arg(z-3-2i) = frac{pi
}{6}$
and $displaystylearg(z-3-4i) = frac{2pi}{3}$.



So I'm first assuming $z=x+iy$, then putting it in the place of $z$ and putting real parts together and imaginary parts together.



Then I'm using $tantheta = frac{text{imaginary part}}{text{real part}}$



$z-3-4i = (x-3)+(y-2)i$



Then,



$tan30° = frac{y-2}{x-3}$



$frac{1}{sqrt3} = frac{y-2}{x-3}$



I'm processing like this and my answer comes as $y=5/2$



And $x= 3+frac{5sqrt3}{2}-2sqrt3$



But the answer key says there's no such $z$ which satisfies this equation.
Is this the wrong way to solve this question, is my answer wrong or the answer key's?







complex-analysis complex-numbers






share|cite|improve this question















share|cite|improve this question













share|cite|improve this question




share|cite|improve this question








edited Nov 22 '18 at 4:37









Kemono Chen

2,621437




2,621437










asked Nov 22 '18 at 3:53









Kaustuv Sawarn

465




465












  • Why are you equating the arguments of the $Arg$'s? Or should that $2$ be a $4$?
    – Michael Burr
    Nov 22 '18 at 4:00






  • 1




    It may be useful to draw a diagram.
    – Kemono Chen
    Nov 22 '18 at 4:04










  • @MichaelBurr i didn't get you. I'm not equating the arguments.
    – Kaustuv Sawarn
    Nov 22 '18 at 4:06










  • @KemonoChen but I get the answer this way, i want to know what am I doing wrong?
    – Kaustuv Sawarn
    Nov 22 '18 at 4:07










  • Your approach is wrong .
    – Akash Roy
    Nov 22 '18 at 4:07


















  • Why are you equating the arguments of the $Arg$'s? Or should that $2$ be a $4$?
    – Michael Burr
    Nov 22 '18 at 4:00






  • 1




    It may be useful to draw a diagram.
    – Kemono Chen
    Nov 22 '18 at 4:04










  • @MichaelBurr i didn't get you. I'm not equating the arguments.
    – Kaustuv Sawarn
    Nov 22 '18 at 4:06










  • @KemonoChen but I get the answer this way, i want to know what am I doing wrong?
    – Kaustuv Sawarn
    Nov 22 '18 at 4:07










  • Your approach is wrong .
    – Akash Roy
    Nov 22 '18 at 4:07
















Why are you equating the arguments of the $Arg$'s? Or should that $2$ be a $4$?
– Michael Burr
Nov 22 '18 at 4:00




Why are you equating the arguments of the $Arg$'s? Or should that $2$ be a $4$?
– Michael Burr
Nov 22 '18 at 4:00




1




1




It may be useful to draw a diagram.
– Kemono Chen
Nov 22 '18 at 4:04




It may be useful to draw a diagram.
– Kemono Chen
Nov 22 '18 at 4:04












@MichaelBurr i didn't get you. I'm not equating the arguments.
– Kaustuv Sawarn
Nov 22 '18 at 4:06




@MichaelBurr i didn't get you. I'm not equating the arguments.
– Kaustuv Sawarn
Nov 22 '18 at 4:06












@KemonoChen but I get the answer this way, i want to know what am I doing wrong?
– Kaustuv Sawarn
Nov 22 '18 at 4:07




@KemonoChen but I get the answer this way, i want to know what am I doing wrong?
– Kaustuv Sawarn
Nov 22 '18 at 4:07












Your approach is wrong .
– Akash Roy
Nov 22 '18 at 4:07




Your approach is wrong .
– Akash Roy
Nov 22 '18 at 4:07










1 Answer
1






active

oldest

votes


















2














$$arg(z-3-2i)=frac{pi}{6}$$ is a line which originate from $(3,2)$ and making an angle of $30^circ$ with positive $x$ axis.



And $$arg(z-3-4i)=frac{2pi}{3}$$ is a line which originate from $(3,4)$ and making an angle of $120^circ$ with positive $x$ axis.



Now drawing These line in $x-y$ Coordinate axis.



You will get no point of Intersection.



So no $z$ which satisfy above these two equations.






share|cite|improve this answer





















  • Okay, but is my approach wrong? If it is, where is it wrong?
    – Kaustuv Sawarn
    Nov 22 '18 at 4:05










  • Actually you have take above line as infinite ray.
    – D Tiwari
    Nov 22 '18 at 4:06











Your Answer





StackExchange.ifUsing("editor", function () {
return StackExchange.using("mathjaxEditing", function () {
StackExchange.MarkdownEditor.creationCallbacks.add(function (editor, postfix) {
StackExchange.mathjaxEditing.prepareWmdForMathJax(editor, postfix, [["$", "$"], ["\\(","\\)"]]);
});
});
}, "mathjax-editing");

StackExchange.ready(function() {
var channelOptions = {
tags: "".split(" "),
id: "69"
};
initTagRenderer("".split(" "), "".split(" "), channelOptions);

StackExchange.using("externalEditor", function() {
// Have to fire editor after snippets, if snippets enabled
if (StackExchange.settings.snippets.snippetsEnabled) {
StackExchange.using("snippets", function() {
createEditor();
});
}
else {
createEditor();
}
});

function createEditor() {
StackExchange.prepareEditor({
heartbeatType: 'answer',
autoActivateHeartbeat: false,
convertImagesToLinks: true,
noModals: true,
showLowRepImageUploadWarning: true,
reputationToPostImages: 10,
bindNavPrevention: true,
postfix: "",
imageUploader: {
brandingHtml: "Powered by u003ca class="icon-imgur-white" href="https://imgur.com/"u003eu003c/au003e",
contentPolicyHtml: "User contributions licensed under u003ca href="https://creativecommons.org/licenses/by-sa/3.0/"u003ecc by-sa 3.0 with attribution requiredu003c/au003e u003ca href="https://stackoverflow.com/legal/content-policy"u003e(content policy)u003c/au003e",
allowUrls: true
},
noCode: true, onDemand: true,
discardSelector: ".discard-answer"
,immediatelyShowMarkdownHelp:true
});


}
});














draft saved

draft discarded


















StackExchange.ready(
function () {
StackExchange.openid.initPostLogin('.new-post-login', 'https%3a%2f%2fmath.stackexchange.com%2fquestions%2f3008723%2fsolve-for-z-which-satisfy-argz-3-2i-frac-pi6-and-argz-3-4i%23new-answer', 'question_page');
}
);

Post as a guest















Required, but never shown

























1 Answer
1






active

oldest

votes








1 Answer
1






active

oldest

votes









active

oldest

votes






active

oldest

votes









2














$$arg(z-3-2i)=frac{pi}{6}$$ is a line which originate from $(3,2)$ and making an angle of $30^circ$ with positive $x$ axis.



And $$arg(z-3-4i)=frac{2pi}{3}$$ is a line which originate from $(3,4)$ and making an angle of $120^circ$ with positive $x$ axis.



Now drawing These line in $x-y$ Coordinate axis.



You will get no point of Intersection.



So no $z$ which satisfy above these two equations.






share|cite|improve this answer





















  • Okay, but is my approach wrong? If it is, where is it wrong?
    – Kaustuv Sawarn
    Nov 22 '18 at 4:05










  • Actually you have take above line as infinite ray.
    – D Tiwari
    Nov 22 '18 at 4:06
















2














$$arg(z-3-2i)=frac{pi}{6}$$ is a line which originate from $(3,2)$ and making an angle of $30^circ$ with positive $x$ axis.



And $$arg(z-3-4i)=frac{2pi}{3}$$ is a line which originate from $(3,4)$ and making an angle of $120^circ$ with positive $x$ axis.



Now drawing These line in $x-y$ Coordinate axis.



You will get no point of Intersection.



So no $z$ which satisfy above these two equations.






share|cite|improve this answer





















  • Okay, but is my approach wrong? If it is, where is it wrong?
    – Kaustuv Sawarn
    Nov 22 '18 at 4:05










  • Actually you have take above line as infinite ray.
    – D Tiwari
    Nov 22 '18 at 4:06














2












2








2






$$arg(z-3-2i)=frac{pi}{6}$$ is a line which originate from $(3,2)$ and making an angle of $30^circ$ with positive $x$ axis.



And $$arg(z-3-4i)=frac{2pi}{3}$$ is a line which originate from $(3,4)$ and making an angle of $120^circ$ with positive $x$ axis.



Now drawing These line in $x-y$ Coordinate axis.



You will get no point of Intersection.



So no $z$ which satisfy above these two equations.






share|cite|improve this answer












$$arg(z-3-2i)=frac{pi}{6}$$ is a line which originate from $(3,2)$ and making an angle of $30^circ$ with positive $x$ axis.



And $$arg(z-3-4i)=frac{2pi}{3}$$ is a line which originate from $(3,4)$ and making an angle of $120^circ$ with positive $x$ axis.



Now drawing These line in $x-y$ Coordinate axis.



You will get no point of Intersection.



So no $z$ which satisfy above these two equations.







share|cite|improve this answer












share|cite|improve this answer



share|cite|improve this answer










answered Nov 22 '18 at 4:03









D Tiwari

5,4132630




5,4132630












  • Okay, but is my approach wrong? If it is, where is it wrong?
    – Kaustuv Sawarn
    Nov 22 '18 at 4:05










  • Actually you have take above line as infinite ray.
    – D Tiwari
    Nov 22 '18 at 4:06


















  • Okay, but is my approach wrong? If it is, where is it wrong?
    – Kaustuv Sawarn
    Nov 22 '18 at 4:05










  • Actually you have take above line as infinite ray.
    – D Tiwari
    Nov 22 '18 at 4:06
















Okay, but is my approach wrong? If it is, where is it wrong?
– Kaustuv Sawarn
Nov 22 '18 at 4:05




Okay, but is my approach wrong? If it is, where is it wrong?
– Kaustuv Sawarn
Nov 22 '18 at 4:05












Actually you have take above line as infinite ray.
– D Tiwari
Nov 22 '18 at 4:06




Actually you have take above line as infinite ray.
– D Tiwari
Nov 22 '18 at 4:06


















draft saved

draft discarded




















































Thanks for contributing an answer to Mathematics Stack Exchange!


  • Please be sure to answer the question. Provide details and share your research!

But avoid



  • Asking for help, clarification, or responding to other answers.

  • Making statements based on opinion; back them up with references or personal experience.


Use MathJax to format equations. MathJax reference.


To learn more, see our tips on writing great answers.





Some of your past answers have not been well-received, and you're in danger of being blocked from answering.


Please pay close attention to the following guidance:


  • Please be sure to answer the question. Provide details and share your research!

But avoid



  • Asking for help, clarification, or responding to other answers.

  • Making statements based on opinion; back them up with references or personal experience.


To learn more, see our tips on writing great answers.




draft saved


draft discarded














StackExchange.ready(
function () {
StackExchange.openid.initPostLogin('.new-post-login', 'https%3a%2f%2fmath.stackexchange.com%2fquestions%2f3008723%2fsolve-for-z-which-satisfy-argz-3-2i-frac-pi6-and-argz-3-4i%23new-answer', 'question_page');
}
);

Post as a guest















Required, but never shown





















































Required, but never shown














Required, but never shown












Required, but never shown







Required, but never shown

































Required, but never shown














Required, but never shown












Required, but never shown







Required, but never shown







Popular posts from this blog

Biblatex bibliography style without URLs when DOI exists (in Overleaf with Zotero bibliography)

ComboBox Display Member on multiple fields

Is it possible to collect Nectar points via Trainline?